How to arrive at $int_{0}^{1} f^3(x) , dx < (int_{0}^{1} f(x) , dx)^2$?












3












$begingroup$


Define $f : [0,1] to mathbb{R} $ differentiable satisfying $f(0)=0$ and $0<f'(x)<1$.



Prove: $displaystyleint_{0}^{1} f^3(x) , dx < left(displaystyleint_{0}^{1} f(x) , dxright)^2$



I found the following answer,




Define $F(x)=displaystyleint_{0}^{x} f^3(t) , dt - left(displaystyleint_{0}^{x} f(t) , dtright)^2$



We have $$F'(x)=f^3(x)-2 left(displaystyleint_{0}^{x} f(t) , dtright)f(x)=f(x)left[f^2(x)-2 left(displaystyleint_{0}^{x} f(t) , dtright)right]$$
and
$$left[f^2(x)-2 left(displaystyleint_{0}^{x} f(t) , dtright)right]'=2f(x)left[f'(x)-1right]$$
from which we can arrive at the conclusion.




But I felt a little confused.



In the past I usually proved the inequality by classical inequalities and something like Taylor expansion to use the condition about derivative.



But this answer is little out of my expectation.



Do we have other proofs or ideas to arrive at $displaystyleint_{0}^{1} f^3(x) , dx < left(displaystyleint_{0}^{1} f(x) , dxright)^2$? And other thoughts about solving these types of issues?



I think when we talk about $left(displaystyleint_{0}^{1} f(x) , dxright)^2 >ldots ,$ it's a little difficult to deal with. I don't quite know what tools to use.










share|cite|improve this question









$endgroup$








  • 2




    $begingroup$
    I get the feeling that it's related to the fact that $sum_{k=1}^n k^3=(sum_{k=1}^n k)^2$. Perhaps this could be used in a proof somehow?
    $endgroup$
    – AlephNull
    Jan 1 at 16:40
















3












$begingroup$


Define $f : [0,1] to mathbb{R} $ differentiable satisfying $f(0)=0$ and $0<f'(x)<1$.



Prove: $displaystyleint_{0}^{1} f^3(x) , dx < left(displaystyleint_{0}^{1} f(x) , dxright)^2$



I found the following answer,




Define $F(x)=displaystyleint_{0}^{x} f^3(t) , dt - left(displaystyleint_{0}^{x} f(t) , dtright)^2$



We have $$F'(x)=f^3(x)-2 left(displaystyleint_{0}^{x} f(t) , dtright)f(x)=f(x)left[f^2(x)-2 left(displaystyleint_{0}^{x} f(t) , dtright)right]$$
and
$$left[f^2(x)-2 left(displaystyleint_{0}^{x} f(t) , dtright)right]'=2f(x)left[f'(x)-1right]$$
from which we can arrive at the conclusion.




But I felt a little confused.



In the past I usually proved the inequality by classical inequalities and something like Taylor expansion to use the condition about derivative.



But this answer is little out of my expectation.



Do we have other proofs or ideas to arrive at $displaystyleint_{0}^{1} f^3(x) , dx < left(displaystyleint_{0}^{1} f(x) , dxright)^2$? And other thoughts about solving these types of issues?



I think when we talk about $left(displaystyleint_{0}^{1} f(x) , dxright)^2 >ldots ,$ it's a little difficult to deal with. I don't quite know what tools to use.










share|cite|improve this question









$endgroup$








  • 2




    $begingroup$
    I get the feeling that it's related to the fact that $sum_{k=1}^n k^3=(sum_{k=1}^n k)^2$. Perhaps this could be used in a proof somehow?
    $endgroup$
    – AlephNull
    Jan 1 at 16:40














3












3








3


1



$begingroup$


Define $f : [0,1] to mathbb{R} $ differentiable satisfying $f(0)=0$ and $0<f'(x)<1$.



Prove: $displaystyleint_{0}^{1} f^3(x) , dx < left(displaystyleint_{0}^{1} f(x) , dxright)^2$



I found the following answer,




Define $F(x)=displaystyleint_{0}^{x} f^3(t) , dt - left(displaystyleint_{0}^{x} f(t) , dtright)^2$



We have $$F'(x)=f^3(x)-2 left(displaystyleint_{0}^{x} f(t) , dtright)f(x)=f(x)left[f^2(x)-2 left(displaystyleint_{0}^{x} f(t) , dtright)right]$$
and
$$left[f^2(x)-2 left(displaystyleint_{0}^{x} f(t) , dtright)right]'=2f(x)left[f'(x)-1right]$$
from which we can arrive at the conclusion.




But I felt a little confused.



In the past I usually proved the inequality by classical inequalities and something like Taylor expansion to use the condition about derivative.



But this answer is little out of my expectation.



Do we have other proofs or ideas to arrive at $displaystyleint_{0}^{1} f^3(x) , dx < left(displaystyleint_{0}^{1} f(x) , dxright)^2$? And other thoughts about solving these types of issues?



I think when we talk about $left(displaystyleint_{0}^{1} f(x) , dxright)^2 >ldots ,$ it's a little difficult to deal with. I don't quite know what tools to use.










share|cite|improve this question









$endgroup$




Define $f : [0,1] to mathbb{R} $ differentiable satisfying $f(0)=0$ and $0<f'(x)<1$.



Prove: $displaystyleint_{0}^{1} f^3(x) , dx < left(displaystyleint_{0}^{1} f(x) , dxright)^2$



I found the following answer,




Define $F(x)=displaystyleint_{0}^{x} f^3(t) , dt - left(displaystyleint_{0}^{x} f(t) , dtright)^2$



We have $$F'(x)=f^3(x)-2 left(displaystyleint_{0}^{x} f(t) , dtright)f(x)=f(x)left[f^2(x)-2 left(displaystyleint_{0}^{x} f(t) , dtright)right]$$
and
$$left[f^2(x)-2 left(displaystyleint_{0}^{x} f(t) , dtright)right]'=2f(x)left[f'(x)-1right]$$
from which we can arrive at the conclusion.




But I felt a little confused.



In the past I usually proved the inequality by classical inequalities and something like Taylor expansion to use the condition about derivative.



But this answer is little out of my expectation.



Do we have other proofs or ideas to arrive at $displaystyleint_{0}^{1} f^3(x) , dx < left(displaystyleint_{0}^{1} f(x) , dxright)^2$? And other thoughts about solving these types of issues?



I think when we talk about $left(displaystyleint_{0}^{1} f(x) , dxright)^2 >ldots ,$ it's a little difficult to deal with. I don't quite know what tools to use.







real-analysis inequality






share|cite|improve this question













share|cite|improve this question











share|cite|improve this question




share|cite|improve this question










asked Jan 1 at 16:18









ZeroZero

592111




592111








  • 2




    $begingroup$
    I get the feeling that it's related to the fact that $sum_{k=1}^n k^3=(sum_{k=1}^n k)^2$. Perhaps this could be used in a proof somehow?
    $endgroup$
    – AlephNull
    Jan 1 at 16:40














  • 2




    $begingroup$
    I get the feeling that it's related to the fact that $sum_{k=1}^n k^3=(sum_{k=1}^n k)^2$. Perhaps this could be used in a proof somehow?
    $endgroup$
    – AlephNull
    Jan 1 at 16:40








2




2




$begingroup$
I get the feeling that it's related to the fact that $sum_{k=1}^n k^3=(sum_{k=1}^n k)^2$. Perhaps this could be used in a proof somehow?
$endgroup$
– AlephNull
Jan 1 at 16:40




$begingroup$
I get the feeling that it's related to the fact that $sum_{k=1}^n k^3=(sum_{k=1}^n k)^2$. Perhaps this could be used in a proof somehow?
$endgroup$
– AlephNull
Jan 1 at 16:40










2 Answers
2






active

oldest

votes


















3












$begingroup$

Clearly $f(x) > 0$ for all $x in langle 0,1]$. As the solution states, we have
$$left[f(x)^2-2 left(displaystyleint_{0}^{x} f(t) , dtright)right]'=2f(x)left[f'(x)-1right] < 0$$
so $f(x)^2 < 2displaystyleint_0^xf(t),dt$ for all $x in langle 0,1]$. Also notice that $left(displaystyleint_{0}^{x} f(t) , dtright)' = f(x)$ so



begin{align}
int_0^1f(x)^3,dx &= int_0^1f(x)^2, f(x),dx \
&< int_0^12 left(displaystyleint_{0}^{x} f(t) , dtright)left(displaystyleint_{0}^{x} f(t) , dtright)',dx \
&= left(displaystyleint_{0}^{x} f(t) , dtright)^2Bigg|_0^1 \
&= left(displaystyleint_{0}^{1} f(t) , dtright)^2
end{align}






If we assume $f in C^1[0,1]$, we can prove the first inequality even easier:

$$f(x)^2 = int_0^x 2f(t),f'(t),dt< 2displaystyleint_0^xf(t),dt$$






share|cite|improve this answer









$endgroup$





















    3












    $begingroup$

    Write $F(x) = int_0^x f(x)dx$ and $I= int_0^1 f(x)dx$. Using integration by parts formula, we get
    $$begin{eqnarray}
    int_0^1 f(x)^3dx &=& (F(x)-F(1))f(x)^2 Big|^{x=1}_{x=0} + 2int_0^1 (F(1)-F(x))f(x)f'(x)dx\
    &<&2int_0^1 (F(1)-F(x))f(x)dx = -(F(1)-F(x))^2Big|^{x=1}_{x=0} = F(1)^2 =I^2.
    end{eqnarray}$$



    Note: However, this calculation is only valid under the assumption that $f$ is absolutely continuous, or $f'$ is Riemann integrable.






    share|cite|improve this answer











    $endgroup$













    • $begingroup$
      I believe this implicitly assumes that $f'$ is integrable. Still, $+1$.
      $endgroup$
      – mechanodroid
      Jan 1 at 19:32












    • $begingroup$
      @mechanodroid Oops, I missed it. Thanks for your valuable comment!
      $endgroup$
      – Song
      Jan 1 at 19:43












    Your Answer








    StackExchange.ready(function() {
    var channelOptions = {
    tags: "".split(" "),
    id: "69"
    };
    initTagRenderer("".split(" "), "".split(" "), channelOptions);

    StackExchange.using("externalEditor", function() {
    // Have to fire editor after snippets, if snippets enabled
    if (StackExchange.settings.snippets.snippetsEnabled) {
    StackExchange.using("snippets", function() {
    createEditor();
    });
    }
    else {
    createEditor();
    }
    });

    function createEditor() {
    StackExchange.prepareEditor({
    heartbeatType: 'answer',
    autoActivateHeartbeat: false,
    convertImagesToLinks: true,
    noModals: true,
    showLowRepImageUploadWarning: true,
    reputationToPostImages: 10,
    bindNavPrevention: true,
    postfix: "",
    imageUploader: {
    brandingHtml: "Powered by u003ca class="icon-imgur-white" href="https://imgur.com/"u003eu003c/au003e",
    contentPolicyHtml: "User contributions licensed under u003ca href="https://creativecommons.org/licenses/by-sa/3.0/"u003ecc by-sa 3.0 with attribution requiredu003c/au003e u003ca href="https://stackoverflow.com/legal/content-policy"u003e(content policy)u003c/au003e",
    allowUrls: true
    },
    noCode: true, onDemand: true,
    discardSelector: ".discard-answer"
    ,immediatelyShowMarkdownHelp:true
    });


    }
    });














    draft saved

    draft discarded


















    StackExchange.ready(
    function () {
    StackExchange.openid.initPostLogin('.new-post-login', 'https%3a%2f%2fmath.stackexchange.com%2fquestions%2f3058613%2fhow-to-arrive-at-int-01-f3x-dx-int-01-fx-dx2%23new-answer', 'question_page');
    }
    );

    Post as a guest















    Required, but never shown

























    2 Answers
    2






    active

    oldest

    votes








    2 Answers
    2






    active

    oldest

    votes









    active

    oldest

    votes






    active

    oldest

    votes









    3












    $begingroup$

    Clearly $f(x) > 0$ for all $x in langle 0,1]$. As the solution states, we have
    $$left[f(x)^2-2 left(displaystyleint_{0}^{x} f(t) , dtright)right]'=2f(x)left[f'(x)-1right] < 0$$
    so $f(x)^2 < 2displaystyleint_0^xf(t),dt$ for all $x in langle 0,1]$. Also notice that $left(displaystyleint_{0}^{x} f(t) , dtright)' = f(x)$ so



    begin{align}
    int_0^1f(x)^3,dx &= int_0^1f(x)^2, f(x),dx \
    &< int_0^12 left(displaystyleint_{0}^{x} f(t) , dtright)left(displaystyleint_{0}^{x} f(t) , dtright)',dx \
    &= left(displaystyleint_{0}^{x} f(t) , dtright)^2Bigg|_0^1 \
    &= left(displaystyleint_{0}^{1} f(t) , dtright)^2
    end{align}






    If we assume $f in C^1[0,1]$, we can prove the first inequality even easier:

    $$f(x)^2 = int_0^x 2f(t),f'(t),dt< 2displaystyleint_0^xf(t),dt$$






    share|cite|improve this answer









    $endgroup$


















      3












      $begingroup$

      Clearly $f(x) > 0$ for all $x in langle 0,1]$. As the solution states, we have
      $$left[f(x)^2-2 left(displaystyleint_{0}^{x} f(t) , dtright)right]'=2f(x)left[f'(x)-1right] < 0$$
      so $f(x)^2 < 2displaystyleint_0^xf(t),dt$ for all $x in langle 0,1]$. Also notice that $left(displaystyleint_{0}^{x} f(t) , dtright)' = f(x)$ so



      begin{align}
      int_0^1f(x)^3,dx &= int_0^1f(x)^2, f(x),dx \
      &< int_0^12 left(displaystyleint_{0}^{x} f(t) , dtright)left(displaystyleint_{0}^{x} f(t) , dtright)',dx \
      &= left(displaystyleint_{0}^{x} f(t) , dtright)^2Bigg|_0^1 \
      &= left(displaystyleint_{0}^{1} f(t) , dtright)^2
      end{align}






      If we assume $f in C^1[0,1]$, we can prove the first inequality even easier:

      $$f(x)^2 = int_0^x 2f(t),f'(t),dt< 2displaystyleint_0^xf(t),dt$$






      share|cite|improve this answer









      $endgroup$
















        3












        3








        3





        $begingroup$

        Clearly $f(x) > 0$ for all $x in langle 0,1]$. As the solution states, we have
        $$left[f(x)^2-2 left(displaystyleint_{0}^{x} f(t) , dtright)right]'=2f(x)left[f'(x)-1right] < 0$$
        so $f(x)^2 < 2displaystyleint_0^xf(t),dt$ for all $x in langle 0,1]$. Also notice that $left(displaystyleint_{0}^{x} f(t) , dtright)' = f(x)$ so



        begin{align}
        int_0^1f(x)^3,dx &= int_0^1f(x)^2, f(x),dx \
        &< int_0^12 left(displaystyleint_{0}^{x} f(t) , dtright)left(displaystyleint_{0}^{x} f(t) , dtright)',dx \
        &= left(displaystyleint_{0}^{x} f(t) , dtright)^2Bigg|_0^1 \
        &= left(displaystyleint_{0}^{1} f(t) , dtright)^2
        end{align}






        If we assume $f in C^1[0,1]$, we can prove the first inequality even easier:

        $$f(x)^2 = int_0^x 2f(t),f'(t),dt< 2displaystyleint_0^xf(t),dt$$






        share|cite|improve this answer









        $endgroup$



        Clearly $f(x) > 0$ for all $x in langle 0,1]$. As the solution states, we have
        $$left[f(x)^2-2 left(displaystyleint_{0}^{x} f(t) , dtright)right]'=2f(x)left[f'(x)-1right] < 0$$
        so $f(x)^2 < 2displaystyleint_0^xf(t),dt$ for all $x in langle 0,1]$. Also notice that $left(displaystyleint_{0}^{x} f(t) , dtright)' = f(x)$ so



        begin{align}
        int_0^1f(x)^3,dx &= int_0^1f(x)^2, f(x),dx \
        &< int_0^12 left(displaystyleint_{0}^{x} f(t) , dtright)left(displaystyleint_{0}^{x} f(t) , dtright)',dx \
        &= left(displaystyleint_{0}^{x} f(t) , dtright)^2Bigg|_0^1 \
        &= left(displaystyleint_{0}^{1} f(t) , dtright)^2
        end{align}






        If we assume $f in C^1[0,1]$, we can prove the first inequality even easier:

        $$f(x)^2 = int_0^x 2f(t),f'(t),dt< 2displaystyleint_0^xf(t),dt$$







        share|cite|improve this answer












        share|cite|improve this answer



        share|cite|improve this answer










        answered Jan 1 at 17:40









        mechanodroidmechanodroid

        28.9k62648




        28.9k62648























            3












            $begingroup$

            Write $F(x) = int_0^x f(x)dx$ and $I= int_0^1 f(x)dx$. Using integration by parts formula, we get
            $$begin{eqnarray}
            int_0^1 f(x)^3dx &=& (F(x)-F(1))f(x)^2 Big|^{x=1}_{x=0} + 2int_0^1 (F(1)-F(x))f(x)f'(x)dx\
            &<&2int_0^1 (F(1)-F(x))f(x)dx = -(F(1)-F(x))^2Big|^{x=1}_{x=0} = F(1)^2 =I^2.
            end{eqnarray}$$



            Note: However, this calculation is only valid under the assumption that $f$ is absolutely continuous, or $f'$ is Riemann integrable.






            share|cite|improve this answer











            $endgroup$













            • $begingroup$
              I believe this implicitly assumes that $f'$ is integrable. Still, $+1$.
              $endgroup$
              – mechanodroid
              Jan 1 at 19:32












            • $begingroup$
              @mechanodroid Oops, I missed it. Thanks for your valuable comment!
              $endgroup$
              – Song
              Jan 1 at 19:43
















            3












            $begingroup$

            Write $F(x) = int_0^x f(x)dx$ and $I= int_0^1 f(x)dx$. Using integration by parts formula, we get
            $$begin{eqnarray}
            int_0^1 f(x)^3dx &=& (F(x)-F(1))f(x)^2 Big|^{x=1}_{x=0} + 2int_0^1 (F(1)-F(x))f(x)f'(x)dx\
            &<&2int_0^1 (F(1)-F(x))f(x)dx = -(F(1)-F(x))^2Big|^{x=1}_{x=0} = F(1)^2 =I^2.
            end{eqnarray}$$



            Note: However, this calculation is only valid under the assumption that $f$ is absolutely continuous, or $f'$ is Riemann integrable.






            share|cite|improve this answer











            $endgroup$













            • $begingroup$
              I believe this implicitly assumes that $f'$ is integrable. Still, $+1$.
              $endgroup$
              – mechanodroid
              Jan 1 at 19:32












            • $begingroup$
              @mechanodroid Oops, I missed it. Thanks for your valuable comment!
              $endgroup$
              – Song
              Jan 1 at 19:43














            3












            3








            3





            $begingroup$

            Write $F(x) = int_0^x f(x)dx$ and $I= int_0^1 f(x)dx$. Using integration by parts formula, we get
            $$begin{eqnarray}
            int_0^1 f(x)^3dx &=& (F(x)-F(1))f(x)^2 Big|^{x=1}_{x=0} + 2int_0^1 (F(1)-F(x))f(x)f'(x)dx\
            &<&2int_0^1 (F(1)-F(x))f(x)dx = -(F(1)-F(x))^2Big|^{x=1}_{x=0} = F(1)^2 =I^2.
            end{eqnarray}$$



            Note: However, this calculation is only valid under the assumption that $f$ is absolutely continuous, or $f'$ is Riemann integrable.






            share|cite|improve this answer











            $endgroup$



            Write $F(x) = int_0^x f(x)dx$ and $I= int_0^1 f(x)dx$. Using integration by parts formula, we get
            $$begin{eqnarray}
            int_0^1 f(x)^3dx &=& (F(x)-F(1))f(x)^2 Big|^{x=1}_{x=0} + 2int_0^1 (F(1)-F(x))f(x)f'(x)dx\
            &<&2int_0^1 (F(1)-F(x))f(x)dx = -(F(1)-F(x))^2Big|^{x=1}_{x=0} = F(1)^2 =I^2.
            end{eqnarray}$$



            Note: However, this calculation is only valid under the assumption that $f$ is absolutely continuous, or $f'$ is Riemann integrable.







            share|cite|improve this answer














            share|cite|improve this answer



            share|cite|improve this answer








            edited Jan 1 at 19:45

























            answered Jan 1 at 18:49









            SongSong

            18.6k21651




            18.6k21651












            • $begingroup$
              I believe this implicitly assumes that $f'$ is integrable. Still, $+1$.
              $endgroup$
              – mechanodroid
              Jan 1 at 19:32












            • $begingroup$
              @mechanodroid Oops, I missed it. Thanks for your valuable comment!
              $endgroup$
              – Song
              Jan 1 at 19:43


















            • $begingroup$
              I believe this implicitly assumes that $f'$ is integrable. Still, $+1$.
              $endgroup$
              – mechanodroid
              Jan 1 at 19:32












            • $begingroup$
              @mechanodroid Oops, I missed it. Thanks for your valuable comment!
              $endgroup$
              – Song
              Jan 1 at 19:43
















            $begingroup$
            I believe this implicitly assumes that $f'$ is integrable. Still, $+1$.
            $endgroup$
            – mechanodroid
            Jan 1 at 19:32






            $begingroup$
            I believe this implicitly assumes that $f'$ is integrable. Still, $+1$.
            $endgroup$
            – mechanodroid
            Jan 1 at 19:32














            $begingroup$
            @mechanodroid Oops, I missed it. Thanks for your valuable comment!
            $endgroup$
            – Song
            Jan 1 at 19:43




            $begingroup$
            @mechanodroid Oops, I missed it. Thanks for your valuable comment!
            $endgroup$
            – Song
            Jan 1 at 19:43


















            draft saved

            draft discarded




















































            Thanks for contributing an answer to Mathematics Stack Exchange!


            • Please be sure to answer the question. Provide details and share your research!

            But avoid



            • Asking for help, clarification, or responding to other answers.

            • Making statements based on opinion; back them up with references or personal experience.


            Use MathJax to format equations. MathJax reference.


            To learn more, see our tips on writing great answers.




            draft saved


            draft discarded














            StackExchange.ready(
            function () {
            StackExchange.openid.initPostLogin('.new-post-login', 'https%3a%2f%2fmath.stackexchange.com%2fquestions%2f3058613%2fhow-to-arrive-at-int-01-f3x-dx-int-01-fx-dx2%23new-answer', 'question_page');
            }
            );

            Post as a guest















            Required, but never shown





















































            Required, but never shown














            Required, but never shown












            Required, but never shown







            Required, but never shown

































            Required, but never shown














            Required, but never shown












            Required, but never shown







            Required, but never shown







            Popular posts from this blog

            Biblatex bibliography style without URLs when DOI exists (in Overleaf with Zotero bibliography)

            ComboBox Display Member on multiple fields

            Is it possible to collect Nectar points via Trainline?